Page 763

Chapter 26

Spotting and Fixing Issues: Organization, Development, and Graphs

Page 764

How Much Do You Know?

Directions: Try the questions below. The “Category” heading in the explanation for each question gives the title of the lesson that covers how to answer it. If you answered the question(s) for a given lesson correctly, you may be able to move quickly through that lesson. If you answered incorrectly, you may want to take your time on that lesson.

  1. Questions 1–11 refer to the following passage.

  2. Genetically Modified Organisms

    Although biotechnology companies and the chronically naïve imagine that there is no danger to be feared from genetically modified foods, they overlook a plethora of evidence indicating that they may be gambling with people’s lives by continuing to interfere and tamper with nature to create these “Frankenfoods.” Potential problems range from the relatively minor—increased possibilities of allergic reactions to certain foods, for instance—to the potentially devastating—the complete skewing of the balance of an ecosystem. All of these factors should be carefully considered before we choose to risk so much for the possibility of a better tomato.

    [1] For example, the cultivation of insect-resistant plants could lead to the reduction or even destruction of certain insect species that naturally feed on those plants. [2] A change in the insect population could have a disastrous impact on certain bird species that rely on the affected insects as their food source. [3] Ecosystems are relatively stable, and the ripple created by genetically altering one variety of soybean will translate into a shock wavePage 765 of unforeseen repercussions in the long term. [4] And alterations in the balance of the bird population could have further-reaching consequences, all the way up the food chain.

    Further, it is likely that there are dangerous impacts on the genetically modified organisms themselves and on those who consume foods produced from genetically modified organisms. Human studies have not been performed, and the possibility that tampering with an organism’s genetic structure could cause far-reaching health consequences for the people who eat genetically modified foods must be prioritized in future scientific research.

    Arguments about the potential for genetic engineering to end world hunger by maximizing the quantity and quality of food grown around the world are based on an essential fallacy: people do not starve because there is a lack of food. People starve because it is more profitable for businesses to let food go to waste than to distribute it to the world’s impoverished and famine-stricken regions. We have plenty of farmland sitting fallow and plenty of food rotting inPage 766 warehouses. And shockingly, the governments in some countries that struggle with hunger may actually thwart food supplies from reaching their citizens. Many of the agribusinesses arguing that genetically modified foods can solve world hunger are the same companies that accept government subsidies now to limit their production of crops in order to avoid flooding the market. These companies are primarily concerned with profit, and whatever lip service they pay to global well-being, the driving force behind genetically modified organisms and foods is profit, not people. In conclusion, the benefits and risks of any new technology must be carefully considered before implementing that technology.

    1. NO CHANGE
    2. insist
    3. hope
    4. think
    1. NO CHANGE
    2. thoughtlessly interfere and casually tamper
    3. interfere by casually tampering
    4. tamper
    1. NO CHANGE
    2. we as a society
    3. those of us who comprise society
    4. the citizens making up our population
  1. Which choice best connects the sentence with the previous paragraph?

    1. NO CHANGE
    2. On the other hand,
    3. Although unlikely,
    4. DELETE the underlined portion and begin the sentence with a capital letter.
  2. Which choice most effectively sets up the information that follows?

    1. NO CHANGE
    2. Humans are tightly connected to ecosystems,
    3. Each ecosystem is an independent entity,
    4. An ecosystem is a delicate thing,
    1. NO CHANGE
    2. must
    3. would
    4. could
  3. To make this paragraph most logical, sentence 4 should be placed

    1. where it is now.
    2. after sentence 1.
    3. after sentence 2.
    4. DELETED from the paragraph.
  4. At this point, the writer is considering adding the following sentence.

    Some studies indicate that certain genetically modified foods have negative effects on the digestive systems and cardiac health of rats that consume those foods in high quantities.

    Should the writer make the addition here?

    1. Yes, because it provides evidence that helps support the main idea of the paragraph.
    2. Yes, because it provides a significant counterargument to the main claim of the paragraph.
    3. No, because it unnecessarily repeats a detail that appears later in the passage.
    4. No, because it shifts the focus of the passage away from the writer’s primary topic.
    1. NO CHANGE
    2. so
    3. or
    4. since
  5. The writer is considering deleting the underlined sentence. Should the sentence be kept or deleted?

    1. Kept, because it provides an alternate explanation for the main claim of the paragraph.
    2. Kept, because it explains background information about the topic of the paragraph.
    3. Deleted, because it contradicts the writer’s main claim in the passage.
    4. Deleted, because it interrupts the flow of the writer’s argument in the paragraph.
  6. Which choice most effectively concludes the paragraph and the passage?

    1. NO CHANGE
    2. It would be nice if we could trust the very companies that could benefit most from the creation of genetically modified organisms.
    3. Unfortunately, those companies affect so many aspects of modern life that we have no choice but to trust them.
    4. Why would we trust our own well-being and that of the planet to companies recklessly pursuing money at the risk of Mother Earth?

Page 767

Check Your Work

  1. B

    Difficulty: Medium

    Category: Development: Precision

    Getting to the Answer: Read the sentence to understand the context in which the underlined word, “imagine,” is used. The writer uses “imagine” to describe how certain groups portray the risks associated with genetically modified organisms. Because the writer views these groups as the opposition, the word needs to convey a stronger sense of resistance to the truth. As written, this part of the sentence is too benign to match the later references to “gambling with people’s lives” and creating “Frankenfoods.” Choice (B) is correct.

  2. D

    Difficulty: Easy

    Category: Organization: Conciseness

    Getting to the Answer: “Interfere” and “tamper” have the same meaning and are used to express a single idea about intentionally changing nature. The correct answer will eliminate one of those words. Choice (D) is correct.

  3. B

    Difficulty: Medium

    Category: Organization: Conciseness

    Getting to the Answer: This question tests wordiness, but pure length is not the only consideration in this type of question. The correct answer choice must fully convey the writer’s intended meaning. As written, the underlined segment is the shortest answer choice, but it is difficult to determine exactly which group of people the “we” is referring to. Choice (B) is correct because it specifies the people included in “we” using the fewest words.

  4. A

    Difficulty: Medium

    Category: Organization: Transitions

    Getting to the Answer: In order to determine how best to connect two parts of a passage, identify the relationship between those parts. The previous paragraph discusses the potential for various risks from genetically modified foods, while this sentence identifies one particular risk: danger to insect species. Since it presents a specific instance of a general group, an example transition word is needed; (A) is correct. There is no contrast relationship, so (B) is incorrect. Choice (C) is incorrect because the writer is concerned about the risks, so downplaying them would contradict the writer’s purpose. Choice (D) would remove the needed connection altogether.

  5. D

    Difficulty: Medium

    Category: Development: Relevance

    Getting to the Answer: The rest of the sentence discusses the possible unforeseen dangers of genetically modified organisms, so the correct answer must introduce the potential for harm. Eliminate (A) and (C) because stability and independence do not effectively set up the idea of potential harm. Choice (B) introduces humans, which are off-topic for the paragraph, so it is also incorrect. Choice (D) describes ecosystems as “delicate” and therefore easily damaged, so it effectively sets up the information that follows and is the correct answer. 

  6. D

    Difficulty: Medium

    Category: Development: Precision

    Getting to the Answer: Although the word “will” is underlined, neither it nor any of the verbs in the answer choices change the grammatical functions of the sentence or cause an error. In a question like this, check the context to make sure the underlined word suits the meaning the writer wants to convey. Throughout the paragraph, the writer uses the verb “could” to indicate the possibility that certain things are possible but not certain—“could lead” and “could have.” Nothing in the paragraph suggests that the writer has shifted from conjecture (indicated by “could”) to either certainty (“will” and “would”) or necessity (“must”). Choice (D) is correct.Page 768

  7. C

    Difficulty: Hard

    Category: Organization: Sentence Placement

    Getting to the Answer: Sentence 4 discusses how changes to the bird population could have further impacts and begins with the continuation word “And.” Logically, it should follow sentence 2, which introduces how changes to the bird population might come about; that way, the paragraph forms a clear logical chain from a change in plants to insects to birds to the larger ecosystem. This also places sentence 3 at the end of the paragraph to sum up with an explanation of the ecology behind the information in the paragraph. The correct answer is (C).

  8. A

    Difficulty: Medium

    Category: Development: Revising Text

    Getting to the Answer: When deciding whether to add a sentence, consider what it would contribute to the purpose and main idea of the paragraph. The topic sentence states that harm from eating genetically modified organisms is “likely,” but the next sentence says that there have not yet been any human studies. The paragraph’s main claim is currently not well supported, and the proposed addition would provide better evidence for it; (A) is correct.

  9. B

    Difficulty: Medium

    Category: Organization: Transitions

    Getting to the Answer: This question is about choosing the appropriate transition word to connect two clauses of a sentence. The first clause is about the lack of human studies on the effect of eating genetically modified organisms, and the second clause identifies the need for such studies given the potential health risks. The ideas in the second clause are clearly a result of the information in the first, so a cause-and-effect transition is needed. Eliminate (A) and (C). Choice (D) is a cause-and-effect transition, but it would reverse the cause-and-effect relationship of the clauses, so it is incorrect. Only (B) correctly conveys the relationship between the ideas in the sentence.

  10. D

    Difficulty: Medium

    Category: Development: Revising Text

    Getting to the Answer: When a question asks whether to delete a sentence, determine what the main idea of the paragraph is and what the sentence in question contributes to it. In this paragraph, the writer argues that people go hungry not just because of lack of food but because of agribusinesses seeking profit. This sentence offers another reason people go hungry, but that reason is not relevant to the writer’s point. In fact, the sentence interrupts the writer’s discussion of how agribusiness contributes to hunger. The correct answer is (D).

  11. D

    Difficulty: Hard

    Category: Development: Introductions and Conclusions

    Getting to the Answer: To effectively conclude a passage, a sentence must align with the writer’s purpose both in the passage as a whole and in the final paragraph. The passage as a whole is a strong argument against GMOs, based primarily on the potential dangers they pose. The final paragraph grows more passionate, criticizing the counterargument raised by advocates for genetically modified organisms. The final sentence must match these arguments. Choice (D) is correct because it suggests that the stakes for the planet are enormous and our misplaced trust would have dire consequences. Choice (A) is too objective, and choices (B) and (C) are too submissive, to match the writer’s critical tone.Page 769

Organization: Conciseness

To answer a question like this:

As the nineteenth century progressed, nursing became increasingly considered “women’s work,” until, at the turn of the 20th century, female nurses began to organize, unofficially excluding men. The American Nursing Association was formed in 1917, and men were not officially permitted to join until 1930. One of first goals of the female-dominated nursing community was to have men denied admission to and excluded from military nursing. Traditionally, non-volunteer military nurses had been exclusively male, but in 1901, the United States Army Nurse Corps was formed exclusively for women.

  1. NO CHANGE
  2. denied to and excluded from
  3. excluded from
  4. denied and excluded from

You need to know this:

A concise sentence does not include any unnecessary words. Phrasing that is wordy is considered stylistically incorrect on the SAT and needs to be revised. Each word must contribute to the meaning of the sentence; otherwise, it should be eliminated.

A redundant sentence says something twice: “The new policy precipitated a crisis situation.” A crisis is a type of situation, so there is no need to include both “crisis” and “situation.” The sentence should be rephrased as, “The new policy precipitated a crisis.” Redundancy is always incorrect on the SAT.


Wordy/Redundant Sentence Concise Sentence
The superb musical score added enhancement to the experience of the play’s development. The superb musical score enhanced the play’s development.
I did not anticipate the surprising, unexpected plot twist.
I did not anticipate the plot twist.
The students increased some of their knowledge of Tuscan architecture.
The students learned about Tuscan architecture.

Page 770

You need to do this:

Choose the most concise grammatically correct option that conveys the writer’s intended meaning. When answering questions about conciseness:

Explanation:

As written, the underlined segment is redundant: “denied admission to” and “excluded from” have the same meaning. Look for a choice that eliminates one of these phrases. Choice (C) is correct.

If conciseness gives you trouble, study the information above and try these Drill questions before completing the following Try on Your Own questions. Eliminate word(s) to make the sentences more concise without losing meaning.

Drill

  1. It is important to carefully consider and think about what kind of college you wish to attend.
  2. Often, a house cat will typically sleep for up to 16 hours per day.
  3. The whole team felt a sense of excited anticipation in the seconds before the whistle blew.
  4. My sister and I couldn’t come to an agreement with each other about what movie we wanted to watch that afternoon.
  5. Noctilucent clouds appear approximately 82 kilometers above Earth’s surface. This is an altitude more than seven times higher than commercial airlines fly.

Organization: Conciseness Drill Answers

Note: These are not the only ways to correct the sentences; your answers may differ.

  1. It is important to carefully consider and think about what kind of college you wish to attend.
  2. Often, a A house cat will typically sleep for up to 16 hours per day.
  3. The whole team felt a sense of excited anticipation in the seconds before the whistle blew.
  4. My sister and I couldn’t come to an agreement with each other about what movie we wanted to watch that afternoon.
  5. Noctilucent clouds appear approximately 82 kilometers above Earth’s surface. This is an altitude, more than seven times higher than commercial airlines fly.

Page 771

Try on Your Own

Directions: Take as much time as you need on these questions. Work carefully and methodically. There will be an opportunity for timed practice at the end of the chapter.

  1. Questions 2–6 refer to the following passage.

    1. NO CHANGE
    2. modern,
    3. presently current,
    4. DELETE the underlined portion.
    1. NO CHANGE
    2. to understand or to comprehend
    3. to understand and comprehend
    4. to understand
    1. NO CHANGE
    2. ambivalence
    3. complexity
    4. ambivalent complexity
    1. NO CHANGE
    2. George and Jane finish their fairy-tale courtship and suddenly find themselves unable to live together,
    3. the fairy-tale courtship of George and Jane is at a finish and they suddenly find themselves unable to live together,
    4. with sudden abruptness George and Jane find themselves unable to live together when they finish their fairy-tale courtship,
  1. Which choice most effectively combines the sentences at the underlined portion?

    1. nod his head sagely, and he will think about
    2. nod his head sagely; he will think about
    3. nod his head sagely, thinking about
    4. nod his head sagely, be thinking about

Page 774

Organization: Transitions

To answer a question like this:

I still remember the magic of walking home under the cold, brittle blue sky, watching the sun strike the glittering blanket laid down by that first snowfall. The world dripped with frosting, and everything was pure and silent. I breathed deeply, enjoying the sting of the icy air in my nostrils, and set off through the trees, listening to the muffled crunch of my footsteps and the chirps of the waking birds. Later, the cars and schoolchildren and mundane lives would turn the wonderland back into dingy slush; the hush would be interrupted by horns and shouts. Indeed, for now, the sparkling, cloistered world was mine alone. I smiled, and for a moment, my mind was still.

  1. NO CHANGE
  2. But
  3. Consequently,
  4. In fact,

You need to know this:

Writers use transitions to show relationships such as contrast, cause and effect, continuation, emphasis, and chronology (order of events). Knowing which words indicate which type of transition will help you choose the correct answer on test day.  


Contrast Transitions Cause-and-Effect Transitions Continuation Transitions Emphasis Transitions Chronology Transitions
although, but, despite, even though, however, in contrast, nonetheless, on the other hand, rather than, though, unlike, while, yet

as a result, because, consequently, since, so, therefore, thus

also, furthermore, in addition, moreover

certainly, in fact, indeed, that is

before, after, first (second, etc.), then, finally

Page 775

You need to do this:

If a transition word is underlined, you must determine the writer’s intended meaning and find the transition that best conveys this meaning. Use the surrounding text to pinpoint the appropriate word.

Explanation:

The transition must logically connect the information and ideas in the sentences before and after the underlined portion. In the sentence before the underlined transition, the author acknowledges that the specialness of the day will later wear off. In the sentences following the transition, the author puts aside thoughts of the coming noise and slush to enjoy the first snowfall. So there is a contrast between the idea that the wonder of the day will fade and the perception of that wonder in the present moment. The only contrast transition among the choices is (B), which is thus the correct answer.

If transitions give you trouble, study the information above and try these Drill questions before completing the following Try on Your Own questions. 

Drill

  1. The train was delayed; (therefore/in addition), we arrived at our destination two hours late.
  2. (Since/Although) the critics agreed that the movie was terrible, I went to see it anyway.
  3. We need to finish our project (consequently/before) we leave this afternoon.
  4. The hiking trail was difficult to navigate; (finally/indeed), state park guidelines recommend that only experienced hikers attempt the climb.
  5. The morphology of the amoeba is more complex than you might expect; (furthermore/in contrast), the mechanism underlying amoeboid motion is still not fully understood.

Organization: Transitions Drill Answers

  1. The train was delayed; therefore, we arrived at our destination two hours late.
  2. Although the critics agreed that the movie was terrible, I went to see it anyway.
  3. We need to finish our project before we leave this afternoon.
  4. The hiking trail was difficult to navigate; indeed, state park guidelines recommend that only experienced hikers attempt the climb.
  5. The morphology of the amoeba is more complex than you might expect; furthermore, the mechanism underlying amoeboid motion is still not fully understood.

Page 776

Try on Your Own

Directions: Take as much time as you need on these questions. Work carefully and methodically. There will be an opportunity for timed practice at the end of the chapter.


  1. Questions 8–12 refer to the following passage.

  2. The Internet and Conversation

    For as long as I can remember, conversation has always struck me as a strange chimera, something that is half two minds exchanging sophisticated ideas and half two dogs barking at each other. I do not find the banalities of small talk comforting, but boring and idiotic. Now I can dispense with it altogether and proceed right to substantive dialogue, it is almost like flying. I can be talking with the closest of friends or a mere acquaintance with a shared interest. Either way, the kinship is there. I don’t feel myself included by smiles, pats on the back, or eye contact so much as by the willingness of a partner to share my ideas or gift me with thoughts of his own. There is nothing more ingratiating than intellectual passion.

    In part, this has to do with my habit of observing the world from my bedroom. As a child, I was frequently ill and forced to stay inside. Although my health is much better now, I still go out much less than most people. After all, I have everything I need inside. From my room, Page 777I have access to people all over the world through online communities. I can talk about medieval literature with a friend in China and later collaborate on a piece of music with a synthesizer virtuoso in Spain. Everyone can be everywhere they want when they want, and every social interaction feels completely comfortable and natural. There is no need for awkward introductions or a graceful exit—people feel free to launch right into what they want to talk about and, when they are done, just sign off with a “TTYL,” or “talk to you later.” Every exchange is relaxed and succinct.

    The Internet facilitates authentic connections among diverse users who share common interests, but empty chatter is far from universal. On certain discussion boards, you can witness opinions stated and arguments debated with an eloquence that people rarely use when speaking, freely sharing knowledge just for the joy of it. I participate in an online Renaissance music discussion group that has a library of original articles that are the product of a master’s thesis. The author gained no monetary reward for the information (which he made available for free) and receives little praise for it outside the community that shares his interest. He posts because he is passionate about his subject; nonetheless, his enthusiasm for the music inspires him to share what he knows with anyone who wants to learn.

    1. NO CHANGE
    2. When
    3. Later
    4. Where
  3. Which choice provides the best transition within the paragraph?

    1. NO CHANGE
    2. I still remember those long afternoons cooped up inside.
    3. I am still careful to eat well and get plenty of rest.
    4. I regret not being able to play outdoors with other children.
  4. Which choice results in the most effective transition to the information that follows in the paragraph?

    1. NO CHANGE
    2. and knowledge of useful text acronyms is nearly universal among online conversationalists.
    3. and those desiring profound discussion can find one at any time of day or night.
    4. DELETE the underlined portion and end the sentence with a period.
  5. Which choice best connects the sentence with the previous paragraph?

    1. NO CHANGE
    2. Though cloistered in a private space, you can now use technology to access virtually any information in the world, but
    3. Internet speak is often maligned as vacuous in its reliance on acronyms and abbreviations, but
    4. DELETE the underlined portion and begin the sentence with a capital letter.
    1. NO CHANGE
    2. subject; lastly,
    3. subject; however,
    4. subject; indeed,

Page 778

Organization: Sentence Placement

To answer a question like this:

[1] It is amazing how little the structure of the American public school system has changed since its inception. [2] Students still change classes according to bells, even though the bell system originated during the days of factories. [3] School is still not in session during the summer, although most students will not use that time to work on farms. [4] Although class and school sizes have varied widely and the curriculum has certainly become varied, the actual system remains surprisingly similar to the way it once was. [5] Despite these idiosyncrasies, however, the American public school system continues to educate the children of this country in a fair and equitable fashion. [6] Without the established structure, the chaotic nature of school would severely inhibit learning.

To make this paragraph most logical, sentence 4 should be placed

  1. where it is now.
  2. before sentence 1.
  3. after sentence 1.
  4. after sentence 6.

You need to know this:

Some organization questions will ask you to check and potentially fix the placement of a sentence within a paragraph (or a paragraph within a passage, though this is rare). Others will ask you for the best place to insert a new sentence. Your approach in both cases should be the same.

You need to do this:

Look for specific clues that indicate the best organization. Common clues include:

Page 779

Explanation:

Sentence 4 argues that the school system has not changed much, so it should precede a sentence that further discusses the unchanged aspects of the school system. If you place sentence 4 after sentence 1, sentences 2 and 3 become examples of specific ways in which the school system has not changed. Moreover, the word “surprising” in sentence 4 echoes the word “amazing” in sentence 1. Sentence 4 also elaborates on sentence 1 by clarifying that the phrase “school system” does not mean only class sizes or curriculum, so sentence 4 fits best after sentence 1 rather than before it.  Choice (C) is correct.Page 780

Try on Your Own

Directions: Take as much time as you need on these questions. Work carefully and methodically. There will be an opportunity for timed practice at the end of the chapter.

  1. Questions 14–18 refer to the following passage.

  2. Earthquakes

    [1]

    Devastating earthquakes in China, Haiti, Chile, Mexico, and elsewhere have caused many to wonder if earthquake activity so far in 2010 is unusual.

    [2]

    Still, earthquake activity has obviously disastrous consequences. As USGS Associate Coordinator for Earthquake Hazards, Dr. Michael Blanpied, explains, “While the number of earthquakes is within the normal range, this does not diminish the fact that there has been extreme devastation and loss of life in heavily populated areas.”

    [3]

    [1] Scientists say 2010 is not showing signs of unusually high earthquake activity. [2] Some years have had as few as six such earthquakes, as in 1986 and 1989, while 1943 had 32, with considerable variability from year to year. [3] Since 1900, an average of 16 magnitude 7 or greater earthquakes—the size that seismologists define as major—have occurred worldwide each year. [4] With six major earthquakes striking in the first four months of this year, 2010 is well within the normal range. [5] From April 15, 2009, to April 14, 2010, there have been 18 major earthquakes, a number also well within the expected variation.

    Page 781

    [4]

    [1] What will happen next in recent earthquake locations? [2] It is unlikely that any of these aftershocks will be stronger than the earthquakes experienced so far, but structures damaged in the previous events could be further damaged and should be treated with caution. [3] Beyond the ongoing aftershock sequences, earthquakes in recent months have not raised the likelihood of future major events; that likelihood has not decreased, either. [4] Aftershocks will continue in the regions around each of this year’s major earthquakes sites. [5] Large earthquakes will continue to occur just as they have in the past.

    [5]

    [1] Though the recent earthquakes are not unusual, they are a stark reminder that earthquakes can produce disasters when they strike populated areas, especially areas where the buildings have not been designed to withstand strong shaking. [2] What can be done to prepare? [3] However, families and communities can improve their safety and reduce their losses by taking actions to make their homes, places of work, schools, and businesses as earthquake-safe as possible.

    [6]

    [1] The Federal Emergency Management Agency (FEMA) recommends specific earthquake preparedness measures, which are especially vital for those who live in earthquake-prone areas. [2] Most importantly, homeowners should check that their homes meet those building codes that are meant to ensure buildings are structurally solid and safe, such as reinforcing masonry walls and securely installing gas appliances. [3] ThePage 782 house should be assessed for furniture placement that could be potentially hazardous during an earthquake, such as unsecured cabinets, and, based on each room’s layout, every family member should learn the safest place to take cover in every part of the house. [4] While we cannot prevent earthquakes, we must take all possible precautions and heed the predictions of scientists.

    Question asks about the previous passage as a whole.

  1. To make this paragraph most logical, sentence 2 should be placed

    1. where it is now.
    2. before sentence 1.
    3. after sentence 3.
    4. after sentence 4.
  1. To make this paragraph most logical, sentence 4 should be placed

    1. where it is now.
    2. before sentence 1.
    3. after sentence 1.
    4. after sentence 2.
  2. Where is the most logical place in this paragraph to add the following sentence?

    Scientists cannot predict the timing of specific earthquakes.

    1. Before sentence 1
    2. After sentence 1
    3. After sentence 2
    4. After sentence 3
  3. Where is the most logical place in this paragraph to add the following sentence?

    FEMA also advises families to create a plan about what to do if an earthquake were to occur while they were at home.

    1. Before sentence 1
    2. After sentence 2
    3. After sentence 3
    4. After sentence 4

Think about the passage as a whole as you answer question 18.

  1. To make the passage most logical, paragraph 2 should be placed

    1. where it is now.
    2. after paragraph 3.
    3. after paragraph 5.
    4. after paragraph 6.

Page 783

Development: Precision

To answer a question like this:

Incredibly, Men Could Not Work As Military Nurses Until After The Korean War. Today, Depending On The Branch Of Service, Anywhere Between 35% And 70% Of Military Nurses Are Men; This Is In Sharp Complement to the civilian world, where an average of 9% of American nurses are men.

  1. NO CHANGE
  2. completion
  3. contour
  4. contrast

You need to know this:

Some questions test your knowledge of the correct word to use in context. You must identify which word(s) best convey the writer’s intended meaning.

Incorrect Correct
The initial reason the students gather in the auditorium is that it is the only location large enough for all of them. The primary reason the students gather in the auditorium is that it is the only location large enough for all of them.
It is common for children to perform the actions of their parents.  It is common for children to mimic the actions of their parents. 
Zeke apologized for overstepping when he walked into the crowded conference room. Zeke apologized for intruding when he walked into the crowded conference room.

You need to do this:

Read the surrounding text to deduce the author’s intended meaning. Then evaluate all four answer choices. Eliminate the answer choices that:

Page 784If one or more of the words among the answer choices are unfamiliar, the process of elimination can still help you get to the correct answer. If you recognize any of the options, decide whether to keep or eliminate them. For the words that remain, use roots, prefixes and suffixes, and word charge to make your decision. If all else fails, trust your instincts and guess; never leave a question blank.

Explanation:

The percentages given are the best clue here: 35–70% of military nurses are men, while only 9% of civilian nurses are men. Those are very different numbers, so the correct word must convey the discrepancy. The only choice that does so is (D)Page 785

Try on Your Own

Directions: Take as much time as you need on these questions. Work carefully and methodically. There will be an opportunity for timed practice at the end of the chapter.

  1. Questions 20–25 refer to the following passage.

  2. Batteries Out in the Cold

    Many people have trouble starting their cars on a cold winter morning. In a cold car, the engine turns over more slowly or sometimes does not turn over at all. Car owners may credit their cold engines, but the real problem is a cold battery.

    A car’s electric starter motor is powered by the car’s battery. When the motor is united to a battery in a circuit, electrons move through the circuit, creating a current. The more current there is in the circuit, the more power there is to turn the starter motor. Decreasing the number of electrons moving through the circuit decreases the current, resulting in a reduction in the amount of power available to the starter motor. Conversely, increasing the number of electrons moving boosts the current, which then increases the amount of power available to the motor.

    Electrons move through a battery as a result of two chemical reactions that occur within the battery, one at each pole. A typical car battery uses lead and sulfuric acid to create these reactions. At the negative pole, lead reacts with sulfate ions in the acid solution around it to form lead sulfate, giving off electrons. At the positive pole, lead oxide reacts with sulfate ions, hydrogen ions, and electrons in the same solution to also form lead sulfate, taking in electrons. The electrons produced at the negative pole flow through the circuit to the positive pole, providing an electric current in the circuit.

    Page 786

    Temperature affects the speed of chemical reactions in two ways. For a chemical reaction to happen, the molecules of the reactants must collide with enough energy to get the reaction going. As the temperature increases, the motion of the molecules increases. The increased motion of the molecules increases the practicality that they will collide and therefore increases the rate of reaction. The amount of energy in the reactants also increases as temperature increases. These factors make it more likely that the molecules in a battery will collide with enough energy to react, thereby increasing the power in the circuit.

    Low temperatures have the opposite effect from high temperatures. The chemicals in the battery react more slowly at low temperatures, due to fewer and less energetic collisions, so fewer electrons move through the circuit. Because of this, a cold battery may not provide enough energy to start a car. A cold car that will not start will need either associative power from a connection to another car’s battery or a source of heat to warm up the car’s own battery and speed up the chemical reactions within it. 

    1. NO CHANGE
    2. critique
    3. accuse
    4. blame
    1. NO CHANGE
    2. correlated
    3. coupled
    4. assimilated
    1. NO CHANGE
    2. regression
    3. retraction
    4. constriction
    1. NO CHANGE
    2. producing
    3. promulgating
    4. asserting
    1. NO CHANGE
    2. process
    3. confirmation
    4. probability
    1. NO CHANGE
    2. auxiliary
    3. optional
    4. extraordinary

Page 787

Development: Relevance

To answer a question like this:

  1. For hundreds of years, the medical community and conventional wisdom held that ulcers were caused by stress. Strong gastric juices would sometimes burn sores through the lining of the stomach or intestines causing widely varied symptoms, including internal bleeding, inflammation, and stomach pain. Doctors reasoned that if patients with ulcers changed their daily habits to reduce the level of tension in their lives, altered their diets to avoid foods that would irritate the stomach, and took medicine to moderate the amount of stomach acid, these ulcers would heal. Although the problem often recurred, no one seriously questioned why. This medical advice remained standard for generations, until Dr. Barry Marshall came along.

    Beginning in the 1980s, Marshall, an Australian physician, hypothesized that at least some ulcers were caused by bacteria that often lie dormant in the human stomach. The international medical community was fascinated by rare gastric phenomena. It was common knowledge, or so Marshall’s colleagues believed, that no microbes could survive for long in the highly acidic environment of the stomach. At medical conferences, the young, unknown Marshall was regarded as at best, a maverick, and at worst, a quack. Over several years, he and his fellow researcher, Dr. J. Robin Warren, attempted to isolate and identify the bacteria that caused ulcers. As is the case with many medical discoveries, their breakthrough came about partly by accident, when they left a culture growing in the lab longer than intended.

    Which choice most effectively sets up other physicians’ negative opinion of Marshall as described later in the paragraph?

    1. NO CHANGE
    2. The international medical community scoffed.
    3. Marshall had attended the University of Western Australia School of Medicine, graduating in 1974.
    4. The stomach produces hydrochloric acid so that the interior of the stomach is highly acidic.

You need to know this:

Some questions will ask you to choose the most relevant information to include at a specific point in a passage. In questions like this, all of the answer choices are grammatically and stylistically correct. Given this fact, your task is to determine which option provides information that is most pertinent to the passage. The correct choice will relate directly to the  surrounding text and will provide one or more of the following:

Note that when the question has a question stem, you are not being tested on conciseness, even when asked about deleting information. When you see a question stem, focus on relevance rather than conciseness.Page 788

You need to do this:

Read a bit of text before and after the underlined portion (at least the entire sentence in which it appears), looking for clues. Eliminate answer choices that: 

Explanation:

The “negative opinion” referred to in the question stem is the statement, later in the paragraph, that Marshall was regarded as “at best, a maverick, and at worst, a quack.” The underlined sentence, as currently written, is irrelevant to the passage, which does not define ulcers as “rare gastric  phenomena,” so eliminate (A) and examine the other choices for a negative opinion. Both (C) and (D) state neutral facts, but the word “scoffed” in (B) indicates that the medical community was unimpressed with, even disdainful of, Marshall. Choice (B) directly relates to the idea that Marshall was perceived as “at worst, a quack” and is correct.

Page 789

Try on Your Own

Directions: Take as much time as you need on these questions. Work carefully and methodically. There will be an opportunity for timed practice at the end of the chapter.

  1. Questions 27–31 refer to the following passage.

  2. James Polk

    For much of his distinguished career, James Knox Polk followed in the footsteps of Andrew Jackson.1 In fact, “Young Hickory’s” policies were very similar to Jackson’s: both men favored lower taxes, championed the frontiersmen, farmers, and workers, and neither was afraid to indulge in Tennessee whiskey. Polk, however, did not share Jackson’s rather fierce temperament; he was instead known for remaining soft-spoken even as he worked energetically toward his goals. Although history will likely always remember the frontier persona of Andrew Jackson, it was Polk who did much more to shape the course of American history.

    The Polk family was poor—James’s father had emigrated from Scotland and arrived in the U.S. South penniless. From an early age, Polk suffered ill health that would turn out to be a lifelong affliction. Despite his physical shortcomings, he was an able student and graduated from the University of North Carolina with honors in 1818. Two years later, Polk was admitted to the bar to practice law, and in 1823, he married Sarah Childress, the daughter of a prominent planter and merchant from Murfreesboro. From there, he was elected to the U.S. House of Representatives in 1825, serving until 1839. Polk was also Speaker of the House from 1835 to 1839.

    Page 790

    After he left Congress to serve as governor of Tennessee in 1839, it became clear that Polk’s political aspirations were high indeed. During the 1844 presidential campaign, a young Abraham Lincoln threw his support behind Whig Henry Clay instead of the Democratic ex-President Martin van Buren. Both men, as part of their platforms, opposed expansionist policies, and neither intended to annex the independent state of Texas or the Oregon Territory. Polk, spurred on by Jackson’s advice, recognized that neither candidate had correctly surmised the feelings of the people, so he publicly announced that, as president, he would do his utmost to acquire Texas and Oregon. Polk was the first “dark horse” in American politics, coming out of nowhere to win the Democratic nomination and the election.

    As the eleventh President of the United States, Polk pursued an agenda of territorial stability and peace with America’s neighbors. First, he reached an agreement with England that divided the Oregon Territory, carving out the present-day states of Washington and Oregon. Polk also quickly annexed Texas and provoked war with Mexico to acquire California and the New Mexico territory. While these achievements were somewhat diminished by controversy from abolitionists who opposed the spread of slavery into new territories, under Polk’s leadership the dream of “manifest destiny” became a reality, and the United States fully extended its borders from the Atlantic to the Pacific.

    1 U.S. president from 1829 to 1837 and War of 1812 hero often referred to as “Old Hickory.”
    1. NO CHANGE
    2. while they agreed on little regarding taxes or the suffrage of frontiersmen, farmers, and workers, both men were known to indulge in Tennessee whiskey.
    3. both men favored lower taxes; championed the frontiersmen, farmers, and workers; and opposed the controversial Bank of the United States.
    4. both men favored lower taxes; championed the frontiersmen, farmers, and workers; and yet they could not agree on the controversial Bank of the United States.
  3. Which choice results in a sentence that best supports the point developed in the paragraph and is consistent with the information in the rest of the passage?
    1. NO CHANGE
    2. he married Sarah Childress.
    3. he was elected as governor of Tennessee.
    4. he was elected to the Tennessee House of Representatives.
  4. Which choice provides the most relevant detail?

    1. NO CHANGE
    2. the leading Democratic candidate was ex-President Martin van Buren and the Whig candidate was Henry Clay.
    3. the issue of slavery’s expansion into new territories began its long stint as the most divisive issue to plague national politics.
    4. both the leading Democratic candidate, ex-President Martin van Buren, and the Whig candidate, Henry Clay, sought to campaign under the banner of “Manifest Destiny” and territorial expansion.
    1. NO CHANGE
    2. Polk, against Jackson’s advice, recognized
    3. Polk failed to recognize
    4. Polk, against the wishes of his advisors, recognized
    1. NO CHANGE
    2. continued expansion of U.S. territory.
    3. an end to slavery in new U.S. territories.
    4. pushing the United States into a role on the international stage.

Page 791

Development: Revising Text

To answer a question like this:

  1. In an effort to demonstrate that ulcers were caused by bacteria, as he and his colleague Warren believed, Dr. Marshall took a bold step. Although the medical community frowns on such potentially dangerous actions, the doctor experimented on himself by deliberately drinking a flask of the bacteria. Over a two-week period, Marshall developed vague, though not disabling, symptoms, and medical tests showed evidence of ulcers and infection.

    In 2005, Marshall’s bold move earned him and Warren the Nobel Prize in Medicine. Their important advance, like many other scientific discoveries in history, was a combination of experimentation, persistence, and luck.

  2. At this point, the writer is considering adding the following sentence.

    Other researchers’ studies later confirmed that Marshall’s and Warren’s findings apply to about 90% of all ulcers, which can now be cured by a short course of antibiotics instead of being temporarily managed by antacids.

    Should the writer make this addition here?

    1. Yes, because the new sentence demonstrates that Marshall was not foolish in drinking the flask of bacteria as he could cure himself with antibiotics.
    2. Yes, because the new sentence provides conclusive support for Marshall’s inference and explains why Marshall and Warren won the Nobel Prize in Medicine.
    3. No, because Marshall was a brilliant scientist, and his findings did not need to be confirmed by other researchers.
    4. No, because Marshall would not have drunk the flask of bacteria had he believed that doing so would cause ulcers and infection.

You need to know this:

These questions often ask about deleting an underlined portion or adding new text. Consider what information the selection provides and whether that information 1) matches the writer’s focus and 2) helps express the purpose of the sentence or paragraph. Consider what the passage might gain or lose if the proposed revision were made. Read the passage both ways—with and without the proposed change—to see which sounds more cohesive. Be sure to read the sentences before and after the proposed revision to best assess the change in context.

The same principle applies if the question asks whether a sentence should be revised: consider what the passage gains or loses as a result of the revision.Page 792

You need to do this:

Explanation:

The big clue for this question comes from the final paragraph, which states that Marshall won the Nobel Prize for Medicine for his discovery that bacteria cause ulcers. The new sentence confirms that this conclusion was correct and reveals that his discovery led to a new treatment for ulcers. This is powerful support for his winning the Nobel Prize for Medicine, so it adds relevant context to the passage. This reasoning matches choice (B), which is the correct answer.Page 793

Try on Your Own

Directions: Take as much time as you need on these questions. Work carefully and methodically. There will be an opportunity for timed practice at the end of the chapter.

  1. Questions 33–37 refer to the following passage.

  2. Jupiter

    As the fifth planet from the Sun in our solar system, Jupiter has inspired fascination and scientific study for centuries. In fact, it was the initial discovery of this colossal, gaseous planet that marked the first time most astronomers seriously considered the possibility that the movement of other planets was not centered around the Earth. More specifically, when Italian astronomer Galileo first viewed Jupiter by telescope, he discovered four large moons in orbit around this enormous planet.  

    The first close look at Jupiter came in 1973, when the unmanned NASA probe Pioneer 10 completed a successful flyby and collected important data regarding the planet’s chemical composition and interiorPage 794 structure. After completing its mission to Jupiter, Pioneer 10 became the first spacecraft to leave the solar system. Designated as one of the gas planets—along with Saturn, Uranus, and Neptune—Jupiter is composed of about 90 percent hydrogen and 10 percent helium and has no solid surface, only varying densities of gas. In fact, very little is known about the interior of Jupiter. When looking at a gas planet like Jupiter, it is really only possible to see the tops of the clouds making up the outermost atmosphere, and probes have been able to penetrate only about 90 miles below this layer. However, after analyzing traces of water and minerals collected from Jupiter’s atmosphere, scientists believe that the planet has a core of rocky material amounting to a mass perhaps as much as 15 times that of Earth.

    Jupiter’s powerful winds are responsible for unique characteristics of the planet. Its high-velocity winds blow in wide bands of latitude, each moving in an alternate direction. Slight chemical and temperature variations result in particular chemical reactions within each band.  Measurements taken by a number of probes indicate that the powerful winds moving these bands can reach speeds exceeding 400 miles per hour and likely extend thousands of miles below Jupiter’s outer atmosphere. The planet can also rotate on its axis in less than half the time it takes the earth to complete a rotation.

    Overall, Jupiter’s size and features have intrigued astronomers of the past and continue to invite scientific investigation today.

  1. The writer is considering revising the underlined portion of the sentence to read:

    and by far the most massive in our solar system,

    Should the writer make this revision here?

    1. Yes, because it establishes an important contrast between Jupiter and the Sun.
    2. Yes, because it provides an example for a detail that appears later in the sentence.
    3. No, because statistics about Jupiter should be placed in a later paragraph.
    4. No, because it contains a fact that contradicts the main claim of the passage.
  2. At this point, the writer is considering adding the following sentence.

    Because these moons seemed to revolve around a planet other than Earth, their discovery provided important evidence for Galileo’s outspoken support of Copernicus’s heliocentric theory of planetary movement.

    Should the writer make this addition here?

    1. Yes, because it explains a detail that was mentioned earlier in the paragraph.
    2. Yes, because it adds to the authority of the passage’s claim by identifying famous scientists who studied Jupiter.
    3. No, because the subject of planetary movement is outside the scope of the passage.
    4. No, because it includes information that is on topic, but irrelevant to the paragraph.
  3. The writer is considering deleting the underlined sentence. Should the writer do this?

    1. No, because it adds an interesting detail about the Pioneer 10 mission.
    2. No, because it provides support for the claim that the mission was successful.
    3. Yes, because it shifts the focus of the passage from Jupiter to the Pioneer 10 mission.
    4. Yes, because it does not include information about when Pioneer 10 left the solar system.
  4. At this point, the writer is considering adding the following sentence.

    These interactions are responsible for creating the array of distinctive, colorful stripes that dominate the planet’s appearance.

    Should the writer make this addition here?

    1. Yes, because it provides support for the claim made in the paragraph’s topic sentence.
    2. Yes, because it provides a useful transition between ideas in the paragraph.
    3. No, because it interrupts the paragraph’s discussion of latitude bands on Jupiter’s surface.
    4. No, because it supplies information that is irrelevant to the main claim of the passage. 
  5. The writer is considering deleting the underlined sentence. Should the sentence be kept or deleted?

    1. Kept, because it provides a relevant detail for the paragraph.
    2. Kept, because it effectively links this paragraph with the conclusion paragraph.
    3. Deleted, because it does not reflect the topic of the overall passage.
    4. Deleted, because it does not support the topic sentence of the paragraph.

Page 795

Introductions and Conclusions

To answer a question like this:

  1. As if malpractice suits and unnecessary bankruptcies were not enough of a problem, lawyers have chosen to increase the burden that they place on society by engineering an excess of increasingly ridiculous product warnings. Why else would a box of sleeping pills be marketed with the cautionary note that consumers may experience drowsiness? Or a cup of coffee be emblazoned with a notice that “THIS PRODUCT MAY BE HOT”? Anyone with common sense will not need to be warned about these possibilities, and anyone without common sense is probably not going to be stopped by a warning label from undertaking a foolish course of action anyway. So honestly, in the long run, the only ones who benefit from these warnings are the lawyers who are paid hundreds of dollars an hour to compose them.

  2. Which choice most effectively establishes the main topic of the paragraph?

    1. NO CHANGE
    2. Each year, effective product warning labels help countless people avoid serious injuries resulting from their use of consumer products.
    3. In recent years, a coalition of lawyers and consumer safety advocates has successfully campaigned to require companies to include safety warnings on product labels.
    4. Product safety warnings are necessary to protect consumers who thoughtlessly use products without first thinking about the possible risks involved.

You need to know this:

Some questions ask you to improve the beginning or ending of a paragraph or passage.

Page 796

You need to do this:

Explanation:

The author introduces the central idea that lawyers are responsible for an excess of product warnings. In the remainder of the paragraph, the author argues that many product warnings are unnecessary and ineffective for consumers and beneficial only to the lawyers who write them. There is no discrepancy between the opening statement and the evidence for it, so (A) is correct.Page 797

Try on Your Own

Directions: Take as much time as you need on these questions. Work carefully and methodically. There will be an opportunity for timed practice at the end of the chapter.

  1. Questions 39–42 refer to the following passage.

  1. Which choice provides the most appropriate introduction to the passage?

    1. NO CHANGE
    2. the skin is not only the human body’s largest organ but also crucial for the body’s healthy functioning.
    3. each square centimeter of human skin has 6 million cells, 5,000 sensory points, and 100 sweat glands.
    4. the skin is an interesting organ of the human body and thus highly studied by biologists.
  2. Which choice most effectively concludes the sentence and paragraph?

    1. NO CHANGE
    2. modifying substances that come into contact with the skin’s surface.
    3. although other organs, such as the liver, detoxify harmful substances with special enzymatic processes.
    4. detoxifying harmful substances with many of the same enzymatic processes the liver uses. 
  3. Which choice is the best introduction to the paragraph?

    1. NO CHANGE 
    2. Another important property of the skin is that it provides our sense of touch.
    3. Although scientists disagree about the total number, we most commonly refer to five human senses.
    4. The central nervous system communicates messages to our brain about our tactile sensations.
  1. Which choice most effectively concludes the passage?

    1. NO CHANGE
    2. Since skin covers the body and is easily visible, it is no wonder that its decoration has important cultural meanings.
    3. However, more important organs do indeed exist and likely deserve more scientific attention than the skin.
    4. Without the skin’s properties, most importantly the sense of touch, life would hardly be worth living.

Page 799

Graphs

To answer a question like this:

  1. Yet perhaps the most fascinating characteristic of this planet is the rotational speed of the entire globe of gas itself. While Earth takes 24 hours to make a full rotation, Jupiter completes a full rotation in less time, an amazingly short period of time for a planet with a diameter roughly 11 times the diameter of our planet. How Jupiter is able to rotate so fast is just one of many mysteries that scientists continue to explore in their efforts to understand our largest neighbor.

    Planets in Our Solar System
    Planet Period of Revolution Around the Sun (1 planetary year) Period of Rotation (1 planetary day) Mass (kg) Diameter (miles)
    Mercury 87.96 Earth days 58.7 Earth days 3.3 × 1023 3,031 miles
    Venus 224.68 Earth days 243 Earth days 4.87 × 1024 7,521 miles
    Earth 365.26 days 24 hours 5.98 × 1024 7,926 miles
    Mars 686.98 Earth days 24.6 Earth hours 6.42 × 1023 4,222 miles
    Jupiter 11.862 Earth years 9.84 Earth hours 1.90 × 1027 88,729 miles
    Saturn 29.456 Earth years 10.2 Earth hours 5.69 × 1026 74,600 miles
    Uranus 84.07 Earth years 17.9 Earth hours 8.68 × 1025 32,600 miles
    Neptune 164.81 Earth years 19.1 Earth hours 1.02 × 1026 30,200 miles
  2. Which choice most accurately reflects the data in the table?

    1. NO CHANGE
    2. in half that time,
    3. in fewer than 10 Earth hours,
    4. in more time than all the other planets,

Page 800

You need to know this:

One or more passages on the Writing and Language section may be accompanied by some sort of informational graphic such as a table, bar chart, or pie chart. If a passage is accompanied by a graphic, at least one question will ask you to revise the passage based on the data presented or will ask you to draw an inference based on the data. Search both the question stem and the answer choices for information that will help you pinpoint the specific parts of the table or graph that apply to the question. The correct answer will:

You need to do this:

Eliminate choices that:

Explanation:

The sentence to which the underlined portion belongs is comparing Earth’s rotational speed to that of Jupiter’s. The relevant information is how much faster Jupiter rotates. Look in the third column of the table for that data to find that Jupiter completes one rotation in 9.84 Earth hours. Compare this information to the answer choices; 9.84 is “fewer than 10,” so (C) is correct.Page 801

Try on Your Own

Directions: Take as much time as you need on these questions. Work carefully and methodically. There will be an opportunity for timed practice at the end of the chapter.

  1. Questions 44–47 refer to the following passage.

  2. Child Expenditures

    A report from the United States Department of Agriculture estimates that the average cost of raising a child born in 2015 until age seventeen is over $230,000. This total includes housing, food, transportation, health care, child care, and education; the overall cost varies considerably from family to family. However, with the average cost of raising a child set at nearly a quarter million dollars, and with additional children in the family raising that financial expenditure accordingly, it becomes clear that parenthood is a major undertaking. When planning a family, future parents should keep financial considerations in mind.

    Even for families that plan for children, costs may be higher than many first-time parents anticipate. For instance, if one parent chooses to spend the first years of the child’s life as a full-time caretaker and homemaker, he or she can lose career momentum and end up making a substantially lower salary than someone with the same background who maintains consistent employment. On the other hand, having both parents in the workforce significantly increases what the family must spend on child care, which constituted an average of 18 percent of the overall costs to raise a child in 2012. While these factors should in no way be construed as a recommendation against having children, they speak to the need for responsible family planning and financial preparation. Page 802

    Expenditures on a Child from Birth Through Age 17: Average Budgetary Component Shares, 1960 versus 2012

    Questions and ask about the pie charts.

  1. The writer wants to provide an accurate and specific description of all the components that are included in the cost of raising a child based on the information in the pie charts. Which choice most effectively accomplishes this goal?
    1. NO CHANGE
    2. basic necessities and other needs;
    3. primary needs such as housing, food, clothing, and health care, in addition to child care, transportation, education, and other assorted costs;
    4. child care, health care, clothing, transportation, housing, and food;
  2. Which choice completes the sentence with accurate data based on the pie charts?

    1. NO CHANGE
    2.  which, combined with education, averaged nearly a fifth of the overall costs to raise a child in 2012.
    3. which, combined with education, costs families considerably more in 2012 than in 1960.
    4. which made up the largest portion of a family’s average expenditures to raise a child in 2012.
  3. Which choice offers an accurate interpretation of the data in the pie charts?

    1. In 1960, health care and transportation together accounted for an average of 20 percent of a budget to raise a child.
    2. Households in 1960 spent an average of 31 percent of their total income on housing costs.
    3. The smallest proportion of an average 2012 budget to raise a child was tied between health care and miscellaneous costs.
    4. Households in 2012 spent, on average, less money per child on clothing than did households in 1960.
  4. Which claim about the cost of raising children is supported by the two pie charts?

    1. Between 1960 and 2012, the percent of a budget spent on food increased.
    2. Between 1960 and 2012, the percent of a budget spent on health care decreased.
    3. Between 1960 and 2012, the percent of a budget spent on transportation experienced the greatest decrease.
    4. Between 1960 and 2012, the percent of a budget spent on child care and education experienced the greatest increase.

Page 803

How Much Have You Learned?

Directions: For testlike practice, give yourself 9 minutes to complete this question set. Be sure to study the explanations, even for questions you got right. They can be found at the end of this chapter.

    1. Questions 48–58 refer to the following passage.

    2. Reefs at Risk

      Coral reefs are essential to water and coastal ecosystems, yet they are currently at great risk. A coral reef is formed by a community of very small plants and animals. These plants and animals are known as algae and polyps. The algae use sunlight to produce their own food for energy and growth. The polyps eat other small animals that come to feed on the algae. Polyps also make a hard substance, called limestone, which eventually builds up to form a reef. Many people enjoy snorkeling and fishing near these reefs.  Coral reefs contain more than one quarter of all marine life and help reduce storm damage to coastal lands. Ultimately, about 10 percent of the world’s coral reefs have already been destroyed, and nearly 60 percent of the remaining reefs face at least a medium level of risk. 

      Page 804

      Fishing and boating are popular sports near coral reefs. People who fish for a living often use explosives to catch the many fish that are attracted to coral reefs, causing significant damage. Boats also destroy reefs with their anchors, yet tourists who swim in coral reefs often break coral off to keep as a souvenir.

      The health of a coral reef depends on having clean water and sunlight, but industrial activities can threaten these simple resources. Oil or chemical spills in the water near the reefs can harm the polyps, and chemical runoff into streams from mines and farms can also destroy the polyps and algae. Further, development along a coast, such as cutting down trees and building roads or parking lots, increases the amount of dirt and sand that washes into the ocean and settles on the bottom. This debris covers the reef and disrupts sunlight from reaching the coral. Without sufficient sunlight, the algae cannot grow, and in turn, the polyps lack the energy needed to produce limestone and build up the reef.

      [1] Marine biologists have found that small crabs living in coral reefs can help prevent the damage caused by coastal development. [2] They remove the particles of dirt and sand that settle on the coral and stop sunlight. [3] The crabs also eat some of the polyps, which would probably suggest that the crabs might also be a threat to the coral. [4] The crabs help the coral survive  but benefit from the relationship as well. [5] The crabs living on the coral have a steady source of food, and the reef provides the crabs with shelter from Page 805predators. [6] However, when the biologists removed crabs from sections of coral, less coral survived than in the sections where the crabs remained.

      The destruction of coral reefs does not have to continue. Recognizing the part that local animals, such as crabs, can play to reduce the amount of damage will help to slow the loss of coral reefs and may provide better ways to protect them.

      (Citation Adapted from Wheeling Jesuit University/NASA-supported Classroom of the Future, “Exploring the Environment: Coral Reefs.”)
  1. Which choice most effectively combines the underlined sentences?

    1. A coral reef is formed by a community of very small plants and animals; these plants and animals are known as algae and polyps.
    2. Very small plants and animals, known as algae and polyps, make up the community that forms a coral reef.
    3. A coral reef is formed by a community of very small plants and animals.
    4. Known as algae and polyps, the community that is called a coral reef is formed by very small plants and animals.
  2. The writer is considering deleting the previous sentence. Should the writer make this change?

    1. Yes, because it introduces the writer’s own opinion and thus does not match the tone of the passage.
    2. Yes, because it contains information that is not related to the main ideas of the paragraph.
    3. No, because it adds an interesting detail about the topic of the passage.
    4. No, because it provides a specific example of why coral reefs are important.
    1. NO CHANGE
    2. Unfortunately,
    3. Consequently,
    4. Inevitably,
  3. Which choice most accurately represents the information in the chart?

    1. NO CHANGE
    2. 36 percent of the surviving reefs face overexploitation.  
    3. more than 6 in 10 reefs have faced threats of some kind or another.
    4. scientists have classified 36 percent of the remaining reefs as “High Risk.” 
  4. Which choice most effectively establishes the paragraph’s central idea?

    1. NO CHANGE
    2. People who participate in activities near coral reefs often cause damage.
    3. Boats are dangerous to the health of coral reefs.
    4. Coral reefs are fragile, and people should be careful around them.
    1. NO CHANGE
    2. and
    3. but
    4. so
    1. NO CHANGE
    2. elusive
    3. committed 
    4. vital
    1. NO CHANGE
    2. delays
    3. blocks
    4. covers
    1. NO CHANGE
    2. suggests
    3. maybe suggests
    4. almost certainly suggests
  5. Which choice results in a sentence that best supports the central idea of the paragraph?

    1. NO CHANGE
    2. but are one of millions of animals that live in coral reefs.
    3. and can be very large or very small in size.
    4. and are found on coral reefs all over the world.
  6. To make this paragraph most logical, sentence 6 should be placed

    1. where it is now.
    2. before sentence 1.
    3. after sentence 2.
    4. after sentence 3.

Page 806

Reflect

Directions: Take a few minutes to recall what you’ve learned and what you’ve been practicing in this chapter. Consider the following questions, jot down your best answer for each one, and then compare your reflections to the expert responses on the following page. Use your level of confidence to  determine what to do next.Page 807

When should you consider selecting “DELETE the underlined portion”?

How can transition words help you determine the most logical placement for a sentence?

Why is relevance important in determining whether a sentence should be revised, added, or deleted?

Name one goal of the introduction of a paragraph/passage. Name one goal of the conclusion of a paragraph/passage.

How can you determine what you need from a graph or table on the SAT Writing and Language section?

Expert Responses

When should you consider selecting “DELETE the underlined portion”?

If the underlined portion is wordy/redundant, does not enhance the meaning of the sentence, or or does not provide clarity, select “DELETE the underlined portion.” Remember that “DELETE the underlined portion” is just as likely as any of the other three choices.

How can transition words help you determine the most logical placement for a sentence?

The kind of transition word that a sentence begins with (contrast, cause and effect, continuation, emphasis, chronology) determines the purpose the sentence should serve within the context. If the sentence does serve that purpose within the context, it is logically placed.

Why is relevance important in determining whether a sentence should be revised, added, or deleted?

If a sentence is missing relevant information, it needs to be revised. If you are going to add a sentence, it should be relevant to the existing information in the passage. If information is not relevant to the passage, it should be deleted.

Name one goal of the introduction of a paragraph/passage. Name one goal of the conclusion of a paragraph/passage.

A good introduction should explain the topic and purpose and include information that will be discussed later in the paragraph/passage. A good conclusion should summarize the topic and purpose and include information that was discussed earlier in the paragraph/passage

How can you determine what you need from a graph or table on the SAT Writing and Language section?

The question stem and the context surrounding where the graph or table is mentioned in the passage are the best clues as to what is important. Always read the question stem and take a look at the answer choices before you start to analyze the graph or table.

Next Steps

If you answered most questions correctly in the “How Much Have You Learned?” section, and if your responses to the Reflect questions were similar to those of the SAT expert, then consider organization, development, and  graphs areas of strength and move on to the next chapter. Come back to these topics periodically to prevent yourself from getting rusty.

If you don’t yet feel confident, review those parts of this chapter that you have not yet mastered. In particular, review common transition words and how they are used in the Organization: Transitions lesson and how to determine if information is relevant in the Development: Relevance lesson. Then try the questions you missed again. As always, be sure to review the explanations closely.Page 808

Answers and Explanations

Organization: Conciseness

  1. Review the Explanation portion of the Organization: Conciseness lesson.
  2. D

    Difficulty: Medium

    Getting to the Answer: Since the previous sentence already identifies that the “distracted reader” is a “member of modern society,” repeating a word that means “modern” is redundant. Deleting the underlined phrase is most concise, so choice (D) is correct.

  3. D

    Difficulty: Easy

    Getting to the Answer: Using two words with essentially the same meaning in the same sentence is redundant. Because “understand” and “comprehend” have the same meaning, the correct answer will eliminate one of those words. Choice (D) is the only answer choice that does so and is correct.

  4. A

    Difficulty: Medium

    Getting to the Answer: “Complexity” and “ambivalence” are not synonyms, so the inclusion of both is not automatically a conciseness issue. Check the surrounding context to see if both words should be included. The purpose of the paragraph is to suggest that the modern story mirrors modern life. In sentence 2, the writer describes the modern story as having “complexity and ambivalence.” It is thus appropriate to repeat the phrase to describe the modern reader’s life, making choice (A) correct. (B) and (C) are incorrect because using only one of the words changes the meaning of the writer’s comparison of the modern story and the modern reader’s life. (D) changes the intended meaning by using “ambivalent” to describe “complexity” rather than the reader’s “life.”

  5. B

    Difficulty: Medium

    Getting to the Answer: Choice (B) is correct because it most concisely expresses the intended meaning by using “George and Jane” as the subject and “finish . . . and . . . find” as the compound predicate. (A) is incorrect because the phrase “come to the completion” is unnecessarily wordy. (C) is incorrect because it expresses the intended meaning less concisely than (B) by including both “George and Jane” and “they” and using the wordy phrase “at a finish.” (D) is incorrect because the words “sudden” and “abruptness” are redundant. 

  6. C

    Difficulty: Hard

    Getting to the Answer: Questions that require you to combine sentences usually are not testing punctuation, so consider issues such as conciseness. Choice (C) most effectively combines the sentences by turning the second sentence into a modifying phrase that provides information about the action in the independent clause. Although (A) and (B) correctly combine the two independent clauses, both are incorrect because repeating the subject (“he”) is unnecessary.

Organization: Transitions

  1. Review the Explanation portion of the Organization: Transitions lesson.
  2. B

    Difficulty: Medium

    Getting to the Answer: A transition word must correctly convey the relationship between the information or ideas it connects. In the sentence before the underlined transition, the author describes small talk as boring. In the following sentence, he compares a conversation without small talk to flying. Since the sentence suggests that small talk can be dispensed with only occasionally, choice (B) is correct. Choices (A) and (C) are incorrect not only because they use inappropriate transition words, but also because they result in run-on sentences.

  3. A

    Difficulty: Medium

    Getting to the Answer: To answer questions about effective transitions within a paragraph, identify the focus of the paragraph both before and after the transition. The first part of the paragraph discusses the reasons the narrator stayed in his room as a child—because he was ill. After the transition, the narrator discusses why he still stays in his room—because he has access to the world through the Internet. The transition needs to show how those ideas are linked. Choice (A) provides a logical transition as written.Page 809

  4. A

    Difficulty: Medium

    Getting to the Answer: This sentence transitions from a discussion of the geographic freedom offered by Internet communication to one about the ease of Internet communication. To complete the transition, the underlined clause needs to relate to ease of communication. The sentence does this effectively as written, so (A) is correct. Choice (D) is incorrect because deleting the underlined portion would remove the transition. Choice (C) is incorrect because it references ideas from a different part of the passage. Choice (B) may be tempting because the sentence following the transition does mention text acronyms, but the writer is focused on ease rather than usefulness or frequency, so (B) is not a correct transition.

  5. C

    Difficulty: Medium

    Getting to the Answer: The end of the previous paragraph describes Internet conversations as “relaxed and succinct;” the remainder of this sentence states that “empty chatter” is not everywhere. The change of opinion from “relaxed and succinct” to “empty chatter” is jarring without a transition, so eliminate (D). The remaining answer choices all contain the contrast word “but,” so the underlined portion must contrast with the writer’s positive opinion about Internet communication. Choice (C) is correct.

  6. D

    Difficulty: Easy

    Getting to the Answer: To determine the correct transition word to use in context, first determine the relationship between the ideas it must connect. The first part of this sentence describes a person as “passionate,” and the second part says that “his enthusiasm . . . inspires him.” The second part elaborates on how his passion motivates him, so a continuation transition is needed to connect the two clauses. The correct answer is (D).

Organization: Sentence Placement

  1. Review the Explanation portion of the Organization: Sentence Placement lesson.
  2. C

    Difficulty: Hard

    Getting to the Answer: Sentence 2 includes some unclear references like “some years” and “such earthquakes,” while also including very specific information about variations in earthquake frequency in particular years. Sentence 3 defines “some years” (since 1900) and “such earthquakes” (major ones) and establishes the average number of earthquakes per year. Sentence 2 would thus fit best after sentence 3 so that its references are more clear and it can elaborate on the established baseline with more specific information. Additionally, this placement results in a general-to-specific and chronological structure for the paragraph, improving its overall logic. Choice (C) is correct.

  3. C

    Difficulty: Medium

    Getting to the Answer: When reordering a sentence within a paragraph, identify the information in the sentence and locate where in the paragraph that information is discussed. The topic of sentence 4 is aftershocks. Aftershocks are also discussed in sentences 2 and 3. However, in sentence 2 the pronoun “these” indicates that aftershocks have been previously discussed in the paragraph, so a sentence about aftershocks prior to sentence 2 is necessary. Thus, the logical place for sentence 4 is before sentence 2. Choice (C) is correct.

  4. C

    Difficulty: Medium

    Getting to the Answer: The transition word “However” at the beginning of sentence 3 indicates that it must contrast with the sentence before it. Sentence 3 is about how to prepare for an earthquake, and sentence 2 asks how to prepare. This does not provide a contrast, but inserting the new sentence before sentence 3 would give a contrast between the inability to precisely predict earthquakes and ways to prepare for them anyway. Placing the new sentence anywhere else in the paragraph would not resolve the logical problem in the paragraph, so (C) is the correct answer.

  5. B

    Difficulty: Medium

    Getting to the Answer: The word “also” in this sentence shows that the sentence needs to follow something else that FEMA advises. Additionally, the paragraph will likely elaborate on how to make a home earthquake plan, so this sentence must come before that information. The correct answer is (B). Placing the new sentence anywhere before sentence 2 would make the “also” logically inconsistent, since sentence 2 contains the first recommendation from FEMA, and placing it anywhere after sentence 3 would put it after the explanation of the plan it introduces.Page 810

  6. B

    Difficulty: Hard

    Getting to the Answer: An effective body paragraph can provide evidence to support the writer’s central idea or introduce a change in the focus of the passage. This paragraph begins with the transition word “still” and establishes a contrast between the number of recent earthquakes and the damage that those earthquakes can cause. The passage also shifts focus from a discussion of earthquake frequency to one of earthquake damage. Therefore, placing this paragraph between the two discussions in paragraph 3 and paragraph 4 makes the most sense. Choice (B) is correct.

Development: Precision

  1. Review the Explanation portion of the Organization: Precision lesson.
  2. D

    Difficulty: Medium

    Getting to the Answer: Establish how the underlined word is used in the sentence; consider the negative or positive tone of the answer choices. The correct word should reflect a negative tone in relation to the problem with the cold engine. “Credit” is a term with a positive connotation, so it’s incorrect. “Critique” means to critically evaluate, which is not the correct meaning in this context. “Accuse” means to charge with a crime, which one wouldn’t do with a car’s engine. “Blame” has the appropriately negative connotation and the correct meaning in this context, so (D) is correct.

  3. C

    Difficulty: Hard

    Getting to the Answer: Establish how the underlined word is used in the sentence based on the surrounding context. The correct word should reflect a connection between the battery and the motor. “United” means made into one entity, so it’s incorrect in this context. “Correlated” means associated with, not connected, so it’s incorrect. “Coupled” means connected and has the correct meaning in this context. “Assimilated” means to absorb into or to have taken on the customs or characteristics of one’s surroundings, so it’s not correct here. (C) is correct.

  4. A

    Difficulty: Medium

    Getting to the Answer: Use the surrounding context to establish how the underlined word is used in the sentence; the following sentence points out that the converse situation is when more electrons result in an increase in power, so it makes sense that fewer electrons result in a decrease in power. “Reduction” means decrease, so it is correct. “Regression” means moving backwards. “Retraction” means pulling back or pulling in. “Constriction” means squeezing or putting under inward pressure. Therefore, (A) is correct.

  5. B

    Difficulty: Medium

    Getting to the Answer: Establish how the underlined word is used in the sentence; the previous lines describe the process by which the battery “makes” electricity. You can predict a similar meaning for the correct choice. “Providing” means giving, not making. “Producing” means making, so it is correct. “Promulgating” means declaring or making known formally, so it doesn’t make sense here, and “asserting” means stating confidently, so it also doesn’t make sense. Choice (B) is correct.

  6. D

    Difficulty: Medium

    Getting to the Answer: Look at how the word is used in the sentence and analyze its meaning. Use context clues to determine which choice is correct. The last sentence of the paragraph says that the molecules will be “more likely” to collide. “Practicality” is a noun meaning feasibility or usefulness and doesn’t make sense here. “Confirmation” means verification, which doesn’t make sense in this context; “process” means method or way, but that is not what’s being described. “Probability” is a noun meaning likelihood, which does make sense in this context. (D) is correct.

  7. B

    Difficulty: Hard

    Getting to the Answer: Use context clues to determine which choice is correct. The sentence says that power will need to be added from another car’s battery, so predict something like “additional.” “Associative” means resulting from association, which is not logical in this context. Auxiliary” means additional or supplementary, and is correct. “Optional” means by choice or not required, which doesn’t make sense here. “Extraordinary” means exceptional or remarkable, which also doesn’t fit the intended meaning. (B) is correct.Page 811

Development: Relevance

  1. Review the Explanation portion of the Development: Relevance lesson.
  2. C

    Difficulty: Medium

    Getting to the Answer: Pay close attention to long lists of evidence to make sure that each component is in line with the central idea and context of the sentence. Choice (C) is correct because it is the only answer choice that both stays focused on matters of policy (as the first part of the sentence mentions) and supports the thesis that Polk and Jackson were in agreement on most points of public policy.

  3. D

    Difficulty: Medium

    Getting to the Answer: Supporting details fit the context of the paragraph and do not contradict details found elsewhere in the passage. They must also logically lead to the information that follows. Select the answer choice that satisfies these guidelines. This paragraph is narrowly concerned with Polk’s origins and early political career. Only (D) fits the context, does not introduce irrelevant information, and does not contradict later details.

  4. B

    Difficulty: Medium

    Getting to the Answer: Remember that supporting evidence needs to focus on and contribute to the central idea. The author’s intent is to introduce the two main candidates in the 1844 election, leading to the following sentence that discusses their opinions of expansionism. Choice (B) is correct because it stays focused on the paragraph’s topic and contributes to the argument that Polk recognized popular support for expansionism that other candidates overlooked or ignored. All other choices introduce details that are either irrelevant or that contradict information elsewhere in the passage.

  5. A

    Difficulty: Medium

    Getting to the Answer: Examine details and parenthetical asides for relevance to the central idea. The goal is to make sure that no contradictions are being introduced into the narrative. No change is necessary because the underlined section touches on two central themes of the passage: Polk’s support for expansionist policies and his close alignment with Jackson’s views. All other choices contradict information in other parts of the passage. Choice (A) is correct.

  6. B

    Difficulty: Medium

    Getting to the Answer: Examine the paragraph for relevance to the central idea. Be sure that the underlined portion is consistent with the information that follows in the rest of the paragraph. Choice (A) is incorrect because the following sentences describe Polk’s expansion of U.S. territory and his provoking a war with Mexico. (B) correctly indicates the idea that Polk’s policies expanded U.S. territory. (C) is inconsistent with the information that follows: there’s a strong implication that Polk’s policies did allow for the expansion of slavery. (D) is incorrect because nothing in the paragraph indicates an expanding international role for the United States. Thus, (B) is correct.

Development: Revising Text

  1. Review the Explanation portion of the Development: Revising Text lesson.
  2. B

    Difficulty: Medium

    Getting to the Answer: In order to decide whether to revise or add information, consider how the change would contribute to the sentence or paragraph. This sentence states that Jupiter interests scientists but does not provide strong reasoning for why. Making the suggested revision would improve the sentence by offering a detail about what makes Jupiter unique that helps to support the sentence’s conclusion. Choice (B) is correct.

  3. A

    Difficulty: Medium

    Getting to the Answer: When considering whether to add a sentence, examine the paragraph to determine whether the sentence to be added would contribute useful information or ideas to it. Before the place the writer is thinking of inserting this sentence, the passage states that the discovery of Jupiter prompted scientists to consider that other planets might not orbit Earth. It then notes that moons orbiting Jupiter were discovered at the same time as the planet. However, the passage does not explain the connection between these statements or support the first statement. The sentence to be added would accomplish both, so (A) is correct.Page 812

  4. C

    Difficulty: Easy

    Getting to the Answer: If you have the option to omit a sentence, think carefully about the author’s topic. If the underlined portion strays from the topic, omit it. The author’s topic is Jupiter. The underlined portion adds information about Pioneer 10 that is irrelevant to the topic. Choice (C) correctly omits the sentence for this reason.

  5. A

    Difficulty: High

    Getting to the Answer: In order to decide whether a sentence should be added, examine the paragraph to see if the sentence supports its purpose. This paragraph is about the winds of Jupiter causing some of its unique features. The sentence in question might not seem related to Jupiter’s winds at first glance, but the sentences before and after it make clear that the “stripes” referred to in the new sentence are the result of bands of wind, since it is these bands that isolate the chemical reactions creating individual colors. Additionally, the colorful, distinctive stripes that the sentence describes are among Jupiter’s unique features, so this sentence supports the paragraph’s topic; (A) is correct.

  6. D

    Difficulty: Medium

    Getting to the Answer: When determining whether to delete a sentence, consider whether it supports the main idea of the passage and the paragraph. This sentence is about Jupiter’s rotation speed. While the passage is about Jupiter, this paragraph is specifically about the winds of Jupiter, so its rotation speed is off topic for the paragraph. Choice (D) is correct because it deletes the sentence for this reason.

  1. Review the Explanation portion of the Introductions and Conclusions lesson.
  2. B

    Difficulty: Medium

    Getting to the Answer: The underlined portion will need to introduce the passage as a whole in addition to fitting well with the rest of its sentence and paragraph. The primary purpose of the passage is to describe the functions of the skin. Choice (B) introduces this topic effectively; the detail about the body’s largest organ connects to the sentence via a contrast with people not thinking about skin as an organ and to the paragraph via an emphasis on the skin’s size. Choice (A) focuses too narrowly on the skin’s communication functions. Choice (C) gives details about the skin’s composition rather than an introduction to the passage. Choice (D) is vague and the mention of biologists is off topic.

  3. D

    Difficulty: Medium

    Getting to the Answer: Pick the answer choice that is in line with the writer’s central idea. In this paragraph, the writer explains how the skin regulates and protects the body. The final sentence is focused on how the skin protects the body, so the underlined portion must be as well. Only (D) correctly centers on the protection the skin offers. Choice (A) is about us protecting our skin rather than vice versa; (B) does not discuss protection; (C) does not discuss the skin.

  4. B

    Difficulty: Hard

    Getting to the Answer: When a question asks for a good introduction sentence, first determine the main idea of the paragraph. This paragraph uses a lot of scientific jargon, but the second sentence refers to “other senses,” and nerves on the body’s surface that receive “tactile stimuli” must refer to the sense of touch. The only answer choice that focuses on the skin and the sense of touch is (B). Choice (A) is too vague to effectively introduce the paragraph. Choice (C) is too broadly focused on all senses and does not mention the skin. Choice (D) does mention touch but discusses the nervous system rather than the skin.

  5. A

    Difficulty: Medium

    Getting to the Answer: Concluding sentences often reassert or summarize the writer’s central idea; therefore, they cannot fundamentally conflict with the writer’s assertions. In this passage, the writer has made clear that the skin is a very important organ that protects the body from a variety of illnesses and disorders, as well as from physical harm. Choice (A) is correct, as it ties preceding points and details together into a coherent conclusion for the writer’s argument.

Graphs

  1. Review the Explanation portion of the Graphs lesson.
  2. Page 813C

    Difficulty: Medium

    Getting to the Answer: The writer wants a specific, comprehensive list of child care costs as presented in the pie graphs. While (A) seems exhaustive, careful reading shows it lacks the categories of clothing and miscellaneous needs; eliminate it. Choice (D) is similarly missing education and miscellaneous expenses. Only (B) includes all of the expense categories in the pie charts, with the added bonus of sorting them by importance for easier comprehension.

  3. B

    Difficulty: Hard

    Getting to the Answer: The sentence brings up “what a family must spend on child care,” and the underlined portion must elaborate with accurate information from the pie charts. According to the charts, “Child Care and Education” represented 18 percent of expenditures on a child in 2012. Choice (B) includes education with child care and correctly describes their combined cost as “nearly a fifth” of total expenses. Choices (A) and (D) omit education, and (D) incorrectly identifies child care as the largest expenditure (rather than housing). Choice (C) may be tempting, since it also includes education, but it inaccurately represents the data: the pie charts show that the percent of total expenditures spent on child care and education has increased, but the charts offer no actual dollar amounts, which makes it impossible to compare those amounts.

  4. A

    Difficulty: Medium

    Getting to the Answer: To answer this type of question, carefully check each answer choice against the information in the graphs until you find the one that is correct. In 1960, health care and transportation represented 6 percent and 14 percent, respectively, of the total and 20 percent together. Choice (A) is correct. Choice (B) is incorrect because the pie graphs represent budgets spent on children, not total household income. (C) is incorrect because clothing constituted the smallest percent. (D) is incorrect because the graphs do not offer dollar amounts, just proportions of budgets.

  5. D

    Difficulty: Medium

    Getting to the Answer: When a question is very broad and does not offer clues as to what part of the graph to examine, use the answer choices to determine where to look. All of the answer choices for this question require a comparison of percents between the two years. The percent spent on food decreased; eliminate (A). The percent spent on health care increased; eliminate (B). The percent spent on transportation decreased, but not as much as the percent spent on food; eliminate (C). The percent spent on child care and education increased from 2 percent to 16 percent, a greater increase than any other category; (D) is correct.

  1. B

    Difficulty: Medium

    Category: Organization: Conciseness

    Getting to the Answer: As written, the sentences contain redundant language. Look for the answer choice that retains the meaning of the two original sentences but is less wordy and redundant. Choice (B) is the correct answer because it contains the same information as the original sentences but in a more concise manner. While (C) is similar, it omits mention of “algae and polyps”; it also uses passive voice, which you should avoid using on the SAT. Choice (D) also includes the necessary information, but it introduces a modifier error, with “Known as algae and polyps” describing “the community” rather than “very small plants and animals.”

  2. B

    Difficulty: Medium

    Category: Development: Revising Text

    Getting to the Answer: Identify the main idea of the paragraph and then determine whether the sentence supports it. Although sentence 3 is related to the main topic of the paragraph, coral reefs, the information in this sentence does nothing to support the main idea: coral reefs’ significance and threatened status. Choice (B) is correct.

  3. B

    Difficulty: Medium

    Category: Organization: Transitions

    Getting to the Answer: Look for a relationship between the ideas in this sentence and the preceding sentences. The previous sentences describe the importance of coral reefs, and this sentence states that they are in danger of disappearing. “Unfortunately” indicates a shift from positive to negative ideas, so it effectively connects these sentences. Choice (B) is the correct answer.Page 814

  4. A

    Difficulty: Medium

    Category: Graphs

    Getting to the Answer: Evaluate the data presented in the graphic that accompanies the passage. Read each answer choice and eliminate those not supported by the data in the graph. Choice (A) is correct. As the chart shows, almost 60 percent of living reefs are in danger: “Medium Risk” 37% + “High Risk” 21% = 58%.

  5. B

    Difficulty: Easy

    Category: Development: Introductions and Conclusions

    Getting to the Answer: The correct answer will briefly describe the main idea of the paragraph and will be supported by the details in the paragraph. Be careful of answer choices like (C) that summarize a detail provided in the paragraph rather than the central idea. Choice (B) best describes the main idea of the paragraph. All of the details presented in the paragraph are related to how people engaging in activities near coral reefs often cause damage.

  6. B

    Difficulty: Easy

    Category: Organization: Transitions

    Getting to the Answer: To determine the best transition word to connect ideas, determine the relationship between the ideas. Boat anchors and people taking souvenirs both damage coral reefs, so the correct answer will show that the ideas are similar. Choice (B) is correct.

  7. D

    Difficulty: Hard

    Category: Development: Precision

    Getting to the Answer: Look for the answer that most clearly conveys the writer’s meaning. The writer states that the reef “depends on” the resources of clean water and sunlight. These resources are necessary to the coral reefs, so (D), “vital,” is the correct answer. While the resources are “simple,” as in (A), that is not the writer’s intended meaning in this context. Similarly, it is a distortion of the writer’s meaning to say that these resources are “elusive,” (B); the writer does not suggest that clean water and sunlight are rare but that the reef’s access to what it needs is threatened.

  8. C

    Difficulty: Medium

    Category: Development: Precision

    Getting to the Answer: Look for the answer that creates the clearest idea within the sentence. The word “blocks” most clearly illustrates how the particles that settle on the coral keep sunlight from reaching the coral. Choice (C) is correct.

  9. B

    Difficulty: Easy

    Category: Organization: Conciseness

    Getting to the Answer: Avoid answers that are grammatically correct but wordy or redundant. Since the word “might” appears later in the sentence, another word that indicates doubt, such as “probably,” (A), or “maybe,” (C), would result in a redundancy. “Suggests” fits grammatically and is the most concise choice, so (B) is the correct answer.

  10. A

    Difficulty: Medium

    Category: Development: Relevance

    Getting to the Answer: Avoid answers that are related to the paragraph topic but do not contribute to the paragraph’s purpose. The paragraph explains how crabs and coral reefs help each other. Choice (A) is the only one that supports this central idea. The other choices offer details about crabs but not about their mutually beneficial relationship with reefs. The correct answer is (A).

  11. D

    Difficulty: Medium

    Category: Organization: Sentence Placement

    Getting to the Answer: In order to determine the best place for a sentence, look for clues in the the sentence and the surrounding paragraph. Sentence 6 states that more coral survived with the crabs than without them, implying that the crabs are good for the coral. Additionally, the sentence starts with the contrast word “however,” so it must contrast with the sentence before it. Thus, the sentence would effectively follow and refute sentence 3’s suggestion that crabs might be damaging the coral; this supports the central idea of the paragraph, that crabs and reefs are good for each other. Choice (D) is the correct answer.